2019 Prelims Analysis | 04 Jun 2019

 Prelims 2019 Question Paper 

 Prelims 2019 Answer Key 

 CSAT 2019 Answer Key 

1. With reference to Mughal India, what is/are the difference/differences between Jagirdar and Zamindar?

  1. Jagirdars were holders of land assignments in lieu of judicial and police duties, whereas Zamindars were holders of revenue rights without obligation to perform any duty other than revenue collection.
  2. Land assignments to Jagirdars were hereditary and revenue rights of Zamindars were not hereditary.

Select the correct answer using the code given below.

(a) 1 only

(b) 2 only

(c) Both 1 and 2

(d) Neither 1 nor 2

Exp: In Mughal India, Jagirdars under Mansabdari system were assigned land in lieu of assigned duties towards the empire such as police,judicial,military; Zamindars too apart from revenue collecting duties had to render military duties and perform Khidmat such as rendering soldiers to fill the ranks of royal army at times of need and offer gifts at the royal court.

Jagirdars were frequently transferred as promotion and demotion was a regular part of empire as loyalties towards Emperor were largely dependent on the rank of jagir/mansab held by a Noble; on the other hand, Zamindars had hereditary rights over revenue as their sway over rural society was overarching in all aspects which rested on paternal and patronage application of rituals.

Source: refer to Class 7 ncert (CH-4) FOR MANSABDAR/JAGIRDAR (pg 51-52) & Class 12 NCERT-P-2 (pg 211-213)

2. With reference to land reforms in independent India, which one of the following statements is correct?

(a) The ceiling laws were aimed at family holdings and not individual holdings.

(b) The major aim of land reforms was providing agricultural land to all the landless.

(c) It resulted in cultivation of cash crops as a predominant form of cultivation.

(d) Land reforms permitted no exemptions to the ceiling limits.

Exp: The Land reform constitutes the most important measures to improve the economic con­dition of agricultural tenants.The Land reforms program in post-Independence India has evolved through different phases and these includes Abolition of Intermediaries, Tenancy reforms, consolidation of holdings and determination of holdings per family, imposition of ceilings with fewer exemptions, and to distribute surplus land among landless peoples. Hence Option (B) is correct.

Covered in Drishti: DLP Book for UPPCS Mains/ Indian Economy-I/Chapter: Land Reforms

3. The Global Competitiveness Report is published by the

(a) International Monetary Fund

(b) United Nations Conference on Trade and Development

(c) World Economic Forum

(d) World Bank

Exp: The Global Competitiveness Report (GCR) is a yearly report published by the World Economic Forum. 

Source: https://www.weforum.org/reports/the-global-competitveness-report-2018

Covered in Drishti: Tarkash Vol I Pg 119

https://www.drishtiias.com/daily-updates/daily-news-analysis/world-economic-forum-annual-meeting-2019

4. Consider the following statements about ‘the Charter Act of 1813’:

  1. It ended the trade monopoly of the East India Company in India except for trade in tea and trade with China.
  2. It asserted the sovereignty of the British Crown over the Indian territories held by the Company.
  3. The revenues of India were now controlled by the British Parliament.

Which of the statements given above are correct?

(a) 1 and 2 only

(b) 2 and 3 only

(c) 1 and 3 only

(d) 1, 2 and 3

Exp: The Charter Act of 1813 ended trade monopoly of East India company in India except for trade in tea and trade in China. Further it also asserted sovereignty of British Crown over the Indian territories held by company. However the Company was to retain the possession of territories and revenues for 20 years more without prejudice to British Crown.

Source: Spectrum modern India page 566-567, BL Grover page 370

5. With reference to Swadeshi Movement, consider the following statements:

  1. It contributed to the revival of the indigenous artisan crafts and industries.
  2. The National Council of Education was established as a part of Swadeshi Movement.

Which of the statements given above is/are correct?

(a) 1 only

(b) 2 only

(c) Both 1 and 2

(d) Neither 1 nor 2

Exp: Swadeshi Movement had its genesis in opposition to the partition of Bengal in 1905. It emphasised on self-reliance thus contributing to revival of indigenous artisans craft and industries. In August 1906 the National Council of Education was established to provide an impetus to program of national education as a part of Swadeshi movement. 

Source: Bipan Chandra page 130-131

6. Consider the following pairs:

Movement/Organization Leader
1. All India Anti-Untouchability League Mahatma Gandhi
2. All India Kisan Sabha Swami Sahajanand Saraswati
3. Self-Respect Movement Naicker E.V. Ramaswami 

Which of the pairs given above is/are correctly matched?

(a) 1 only

(b) 1 and 2 only

(c) 2 and 3 only

(d) 1, 2 and 3

Exp: Mahatma Gandhi set up All India Anti-Untouchability league in 1932 which later transformed into Harijan Sevak Sangh. All India Kisan Sabha was founded by Swami Sahajanand Saraswati in 1936 to spearhead peasant movements in colonial India. E V Ramaswamy Naicker started Self respect movement against untouchability in 1925.

Source: Spectrum modern India pg 438, Bipan Chandra pg 344, Ncert 8 page 119

7. Which one of the following is not a Harappan site?

(a) Chanhudaro

(b) Kot Diji

(c) Sohgaura

(d) Desalpur

Exp: Kot Diji (now in Pakistan) was an early Harappan site. Chanhudaro (Pakistan) and Desalpur (Gujarat) are mature Harappan sites.

Source: Upinder Singh page146

8. In which of the following relief sculpture inscriptions is ‘Ranyo Ashoka’ (King Ashoka) mentioned along with the stone portrait of Ashoka?

(a) Kanganahalli

(b) Sanchi

(c) Shahbazagrhi

(d) Sohgaura

Exp: Kanaganahalli in Karnataka is the site with an inscription in Brahmi script reading Ranyo Ashoka and a sculpture of King Ashoka.

Source: Upinder Singh page 333

9. Consider the following:

  1. Deification of the Buddha
  2. Treading the path of Bodhisattvas
  3. Image worship and rituals

Which of the above is/are the feature/ features of Mahayana Buddhism?

(a) 1 only

(b) 1 and 2 only

(c) 2 and 3 only

(d) 1, 2 and 3

Exp: Mahayana Buddhism promoted devotional support, Deification of Buddha, icon worship and rituals and treading path of Bodhisattvas.

Source: Ncert 12th pg 103, Ncert 6th pg 103

Covered in Drishti: https://www.drishtiias.com/to-the-points/paper1/buddhism-in-india

10. With reference to forced labour (Vishti) in India during the Gupta period, which one of the following statements is correct?

(a) It was considered a source of income for the State, a sort of tax paid by the people.

(b) It was totally absent in the Madhya Pradesh and Kathiawar regions of the Gupta Empire.

(c) The forced labourer was entitled to weekly wages.

(d) The eldest son of the labourer was sent as the forced labourer.

Exp: Vishti was considered as a source of income for the state, a sort of tax by the people.

Source: Upinder Singh page 508

11. Building ‘Kalyaana Mandapas’ was a notable feature in the temple construction in the kingdom of

(a) Chalukya

(b) Chandela

(c) Rashtrakuta

(d) Vijayanagara

Exp: In the Vittala Temple of Hampi, pavilions or mandapas that were meant to celebrate the marriages of dieties were known as Kalyan Mandapa. 

Source: http://www.asihampiminicircle.in/vitthala-temple/%20ASI%20website

12. Consider the following statements:

  1. In the revenue administration of Delhi Sultanate, the in-charge of revenue collection was known as ‘Amil’.
  2. The lqta system of Sultans of Delhi was an ancient indigenous institution.
  3. The office of ‘Mir Bakshi’ came into existence during the reign of Khalji Sultans of Delhi.

Which of the statements given above is/are correct?

(a) 1 only

(b) 1 and 2 only

(c) 3 only

(d) 1, 2 and 3

Exp: In the Delhi Sultanate, the task of collecting revenue directly from peasant and measurement of land rested on Amils.

Iqta system evolved in West Asia significantly in Persia

Mir Bhakshi was the head of militiary department in Mughal India whereas in Delhi Sultanate, Ariz under Diwan-i-arz (set up by Balban) was improved by Alauddin Khalji as he introduced Dagh system which was aimed to improve Horse quality as well as eliminate fake numbering. 

Source: Satish Chandra History of Medieval India (800-1700) pg 74, Medieval India old Ncert pg 96, 201, 83

13. Consider the following statements:

  1. Saint Nimbarka was a contemporary of Akbar.
  2. Saint Kabir was greatly influenced by Shaikh Ahmad Sirhindi.

Which of the statements given above is/are correct?

(a) 1 only

(b) 2 only

(c) Both 1 and 2

(d) Neither 1 nor 2

Exp: Nimbarka was a 13th century philosphical Saint whereas Akbar's time period was 1556-1605;Saint Kabir was 15th century poet whereas Sirhindi was in 17th century.

Source: Ncert 12th pg 167

https://www.britannica.com/biography/Nimbarka-Indian-philosopher

14. With reference to the British colonial rule in India, consider the following statements:

  1. Mahatma Gandhi was instrumental in the abolition of the system of ‘indentured labour
  2. In Lord Chelmsford’s ‘War Conference’, Mahatma Gandhi did not support the resolution on recruiting Indians for World War.
  3. Consequent upon the breaking of Salt Law by Indian people, the Indian National Congress was declared illegal by the colonial rulers.

Which of the statements given above are correct?

(a) 1 and 2 only

(b) 1 and 3 only

(c) 2 and 3 only

(d) 1, 2 and 3

Exp: In South Africa, Gandhi in opposition to compulsory registration of Indian population under Asiatic Law Amendment Act of the Transvaal government organised non-violent protest which led to passing of Indian Relief Act 1914- which eliminated extra tax on Indian citizens who had not renewed their indentures and recognised validity of Indian marriages.

In the Young India article dated 27 Nov ,1920 - “To Every Englishmen In India”, Gandhi quoted his recruiting campaign which was carried out in Kaira District (Gujarat) in support of British Cause in World War I according to the promise made in War Conference of Delhi.

As the Salt March ended in month of April 1930, between June and August several Congress Working Committees and leaders were arrested and Congress as an political was declared illegal by the British Administration.

Source: (Source: Bipan Chandra-pg 279) http://www.bb`c.com/travel/story/20190325-the-birthplace-of-gandhis-peaceful-protest

http://www.gandhi-manibhavan.org/eduresources/article6.htm

Bipin Chandra pg-279

15. With reference to Indian National Movement, consider the following pairs:

Person Position held
1. Sir Tej Bahadur Sapru President, All India  Liberal Federation
2. K.C. Neogy Member, The Constituent Assembly
3. P.C. Joshi General Secretary, Communist Party of India

Which of the pairs given above is/are correctly matched?

(a) 1 only

(b) 1 and 2 only

(c) 3 only

(d) 1, 2 and 3

Exp: Tej Bahadur Sapru was a prominent member of Liberal Party of India and the President of All India Liberal Federation. KC Neogy was a memeber of Constituent Assembly and also the Chairman of First Finance Commision. P.C Joshi was the General Secretary of Communist Party of India. 

16. With reference to Mian Tansen, which one of the following statements is not correct?

(a) Tansen was the title given to him by Emperor Akbar.

(b) Tansen composed Dhrupads on Hindu gods and goddesses.

(c) Tansen composed songs on his patrons.

(d) Tansen invented many Ragas.

Exp: Tansen was the title given by Raja Vikramjit of Gwalior. Tansen was a court musician in the darbar of Raja Ramachandra of Bandavagarh (Rewa).

Source: https://www.thehindu.com/entertainment/music/the-legend-of-mian-tansen/article22893454.ece

17. Who among the following Mughal Emperors shifted emphasis from illustrated manuscripts to album and individual portrait?

(a) Humayun

(b) Akbar

(c) Jahangir

(d) Shah Jahan

Exp: During Akbar's reign many manuscripts were illustrated, the prominent being Razmnamah, the Persian translation of Mahabharata. It was during Jahangir's reign that manuscripts became less important and individual portraits became more prominent.

Source: Pearson Indian history Manual, NCERT 12th Kings and Chronicles

18. Which one of the following National Parks lies completely in the temperate alpine zone?

(a) Manas National Park

(b) Namdapha National Park

(c) Neora Valley National Park

(d) Valley of Flowers National Park

Exp: The Valley of Flowers national park located in the Chamoli district of Uttarakhand and is known for its meadows of endemic alpine flowers and the variety of flora.Situated approximately 3255m above sea level, this splendid park is spread across 87.5 sq km of land.

The valley has three sub-alpine between 3,200m and 3,500m which is the limit for trees, lower alpine between 3,500m and 3,700m, and higher alpine above 3,700m.

The rich diversity of species reflects the valley's location within a transition zone between the Zanskar and Great Himalayas ranges to the north and south, respectively, and between the Eastern Himalaya and Western Himalaya flora.

Source: http://forest.uk.gov.in/pages/view/159-valley-of-flowers-national-park-

19. Atal Innovation Mission is set up under the

(a) Department of Science and Technology

(b) Ministry of Labour and Employment

(c) NITI Aayog

(d) Ministry of Skill Development and Entrepreneurship

Exp: AlM's objectives are to create and promote an ecosystem of innovation and entrepreneurship across the country at school, university, research institutions, MSME and industry levels. The Mission has been set up under NITI Aayog, in accordance with the Hon'ble Finance Minister's declaration in the 2015 Budget Speech.

Source: http://pib.nic.in/newsite/PrintRelease.aspx?relid=189295

Covered in Drishti: Tarkash Vol I Pg 56, 75, 223

https://www.drishtiias.com/daily-updates/daily-news-analysis/Important%20Facts%20for%20Prelims%20(13th%20October%202018)

https://www.drishtiias.com/summary-of-important-reports/niti-aayog-annual-report-2017-18

https://www.drishtiias.com/daily-updates/daily-news-analysis/Important%20Facts%20for%20Prelims%20(13th%20October%202018)

20. On 21st June, the Sun

(a) does not set below the horizon at the Arctic Circle

(b) does not set below the horizon at Antarctic Circle

(c) shines vertically overhead at noon on the Equator

(d) shines vertically overhead at the Tropic of Capricorn

Exp: 21st June is the longest day of the year as countries lying to the north of the equator, including India, witness ‘Summer Solstice’.The ‘summer solstice’ occurs exactly when the earth’s axial tilt is most inclined towards the sun at its maximum of 23 degrees 26’.During ‘summer solstice’, the Northern Hemisphere witnesses its longest day of the year while the Southern Hemisphere sees its shortest day.

On the June solstice, the Earth’s North Pole is at its maximum tilt toward the sun. At this time, the sun appears directly overhead at 23.5 degrees north latitude, along the Tropic of Cancer.

Around the Arctic Circle, the location of sunrise and sunset start to converge in the northern sky, until eventually the sun never sinks below the horizon. Hence, the appearance of the midnight sun.

21. Which one of the following groups of plants was domesticated in the ‘New World’ and introduced into the 'Old World'?

(a) Tobacco, cocoa and rubber

(b) Tobacco, cotton and rubber

(c) Cotton, coffee and sugarcane

(d) Rubber, coffee and wheat

Exp: "New World" means the Americas which were discovered by European Powers such as Spain, Portuguese and French; Tobacco, Cocoa and Rubber were native crops of Americas and were introduced in the Old World (Asia). 

22. Consider the following statements:

  1. Asiatic lion is naturally found in India only.
  2. Double-humped camel is naturally found in India only.
  3. One-horned rhinoceros is naturally found in India only.

Which of the statements given above is/are correct?

(a) 1 only

(b) 2 only

(c) 1 and 3 only

(d) 1, 2 and 3

Exp: Asiatic lions that once ranged from Persia (Iran) to Palamau in Eastern India were almost driven to extinction by indiscriminate hunting and habitat loss. A single population of less than 50 lions persisted in the Gir forests of Gujarat by late 1890's. With timely and stringent protection offered by the State Government and the Center Government, Asiatic lions have increased to the current population of over 500 numbers. Hence, statement 1 is correct. The double-hump camel is a native of Gobi desert, and is found on a vast expanse of cold-desert areas across Mongolia, China, Kazakhstan, Turkmenistan, Uzbekistan and parts of Afghanistan. Hence, statement 2 is NOT correct. The populations of one horned rhinoceros have increased to around 3,500 rhinos in northeastern India and the Terai grasslands of Nepal. Hence, statement 3 is NOT correct.

Source: http://pib.nic.in/newsite/PrintRelease.aspx?relid=186688

https://www.thehindubusinessline.com/specials/india-interior/two-humps-one-job-in-nubra-valley/article7900544.ece

https://www.worldwildlife.org/species/greater-one-horned-rhino

Covered in Drishti: https://www.drishtiias.com/daily-updates/daily-news-analysis/important-facts-for-prelims-12th-march-2019

https://www.drishtiias.com/daily-updates/daily-news-analysis/asiatic-lion-conservation-project

23. Consider the following pairs:

Famous place River
1. Pandharpur Chandrabhaga
2. Tiruchirappalli Cauvery
3. Hampi Malaprabha

Which of the pairs given above are correctly matched?

(a) 1 and 2 only

(b) 2 and 3 only

(c) 1 and 3 only

(d) 1, 2 and 3

Exp: Pandharpur is a holy place of Shri.Vitthal and Shri.Rukmini. It is also known as the Southern Kashi of India and Kuldaivat of Maharashtra State. The Chandrabhaga (Bhima) river flows through the City.Tiruchirappalli, situated on the banks of the river Cauvery is the fourth largest city in Tamil Nadu. It was a citadel of the early Cholas which later fell to the Pallavas.

Hampi is a UNESCO World Heritage Site in India located near Hospet town in the Karnataka state, on south bank of Tungabhadra river.

Source: https://solapur.gov.in/en/tourist-place/pandharpur/

24. In a given year in India, official poverty lines are higher in some States than in others because

(a) poverty rates vary from State to State

(b) price levels vary from State to State

(c) Gross State Product varies from State to State

(d) quality of public distribution varies from State to State

Exp: These poverty lines would vary from State to State because of inter-state price differentials

Source: http://planningcommission.nic.in/news/pre_pov2307.pdf

25. In the context of which of the following do some scientists suggest the use of cirrus cloud thinning technique and the injection of sulphate aerosol into stratosphere?

(a) Creating the artificial rains in some regions

(b) Reducing the frequency and intensity of tropical cyclones

(c) Reducing the adverse effects of solar wind on the Earth

(d) Reducing the global warming

Exp: Geo-engineering approach to reversing global warming – ‘The intentional reduction of the coverage and optical thickness of high-level cirrus cloud could potentially reduce global warming by modifying the longwave radiative effect of cirrus clouds.’

Source: https://indianexpress.com/article/technology/science/a-cocktail-geo-engineering-approach-to-reversing-global-warming-4805937/

https://www.dailymail.co.uk/sciencetech/article-6364967/Could-thinning-clouds-combat-climate-change-Scientists-propose-radical-plan.html

Covered in Drishti: Tarkash Vol I Pg 268; Tarkash Vol II Pg 125

https://www.drishtiias.com/daily-updates/daily-news-analysis/atmospheric-geoengineering-experiment-to-curb-global-warming

26. In the context of which one of the following arc the terms ‘pyrolysis and plasma gasification’ mentioned?

(a) Extraction of rare earth elements

(b) Natural gas extraction technologies

(c) Hydrogen fuel-based automobiles

(d) Waste-to-energy technologies

Exp: In developing Three-Year Action Agenda (2017-18 to 2019-20), the Niti Aayog has drawn a broader framework for addressing the issue of municipal solid waste (MSW). The solution being suggested by the Action Agenda is twin-fold: waste-to-energy incinerators for bigger municipalities and composting method of waste disposal for small towns and semi-urban areas. While proposing incineration as a solution, the Niti Aayog has also assessed the benefit-cost ratio of thermal pyrolysis and plasma gasification technologies.

Covered in Drishti: https://www.drishtiias.com/to-the-points/paper3/biofuels-1

27. Which of the following are in Agasthyamala Biosphere Reserve?

(a) Neyyar, ldl Peppara and Shendurney Wildlife Sanctuaries; and Kalakad Mundanthurai Tiger Reserve

(b) Mudumalai, Sathyamangalam and Wayanad Wildlife Sanctuaries; and Silent Valley National Park

(c) Kaundinya, Gundla Brahmeswaram and Papikonda Wildlife Sanctuaries; and Mukurthi National Park

(d) Kawal and Sri Venkateswara Wildlife Sanctuaries; and Nagarjunasagar-Srisailam Tiger Reserve

Exp: The agasthyamalai biosphere is located in the Western Ghats in the south of the country. Consisting mostly of tropical forest, the site is home to 2,254 species of higher plants including about 400 that are endemic. It is also a unique genetic reservoir of cultivated plants, in particular cardamom, jamune, nutmeg, pepper and plantain. Three wildlife sanctuaries, Shendurney, Peppara and Neyyar, are located in the site, as well as the Kalakad Mundanthurai Tiger reserve.

Source: http://www.unesco.org/new/en/natural-sciences/environment/ecological-sciences/biosphere-reserves/asia-and-the-pacific/india/agasthyamala/

Covered in Drishti: https://www.drishtiias.com/daily-updates/daily-news-analysis/important-facts-for-prelims-15th-january-2019

28. Consider the following statements:

  1. Some species of turtles are herbivores.
  2. Some species of fish are herbivores.
  3. Some species of marine mammals are herbivores.
  4. Some species of snakes viviparous.

Which of the statements given above arc correct?

(a) 1 and 3 only

(b) 2, 3 and 4 only

(c) 2 and 4 only

(d) 1, 2, 3 and 4

Exp: Green sea turtles eat seagrasses and algae, though juveniles snack on crabs, sponges, and jellyfish. Parrotfish are algae eaters. They obtain the algae by ripping small chunks of coral from a reef. The fish's grinding teeth, located in its throat, pulverize the coral. The teeth allow the fish to extract the algae from the center of the polyp inside.Manatees, sometimes called sea cows, are large mammals that live in warm sea waters. They live in shallow coastal areas and feed on sea vegetation. Snakes that are viviparous nourish their developing young through a placenta and yolk sac, something that is highly unusual among reptiles.

Source: https://sciencing.com/list-herbivores-ocean-8599405.html

29. Consider the following pairs

Wildlife Naturally found in
1. Blue-finned Mahseer Cauvery River
2. Irrawaddy Dolphin Chambal River
3. Rusty-spotted Cat Eastern Ghats

Which of the pairs given correctly matched?

(a) 1 and 2 only

(b) 2 and 3 only

(c) 1 and 3 only

(d) 1, 2 and 3

Exp: The Western Ghats region is famous for freshwater biodiversity. Mahseer, belonging to the genus Tor, is considered a prized sports fish of great cultural value.In the Cauvery the Mahseer community comprises two varieties—a “blue-finned” fish and an “orange-finned, hump-backed” one. Hence, pair 1 is correctly matched. Irrawaddy dolphins are distributed in shallow, near-shore tropical and subtropical marine waters. They are primarily found in estuaries and semi-enclosed water bodies such as bays and sounds, usually close to mangrove forests. Hence, pair 2 is NOT correctly matched. Forests of the Eastern Ghats are home to some endangered and lesser-known mammal species like the Fishing Cat, Rusty Spotted Cat, Indian Pangolin etc. Hence, pair 3 is correctly matched.

Source: https://www.downtoearth.org.in/news/iconic-indian-fish-on-the-verge-of-extinction-says-study-49802

http://wwf.panda.org/knowledge_hub/endangered_species/cetaceans/about/irrawaddy_dolphin/

https://www.thehindu.com/news/national/andhra-pradesh/road-ecology-the-road-less-travelled/article24585647.ece

Covered in Drishti: Tarkash Vol II Pg 153, 155

30. Why is there a great concern about the ‘microbeads' that are released into environment?

(a) They are considered harmful to marine ecosystems.

(b) They are considered to cause skin cancer in children.

(c) They are small enough to be absorbed by crop plants in irrigated fields.

(d) They are often found to be used as food adulterants.

Exp: Microbeads are small, solid, manufactured plastic particles that are less than 5mm and don’t degrade or dissolve in water. They may be added to a range of products, including rinse-off cosmetics, personal care and cleaning products. They are considered harmful to marine and aquatic ecosystem.

Source: https://www.environment.gov.au/protection/waste-resource-recovery/plastics-and-packaging/plastic-microbeads

Covered in Drishti: Tarkash Vol I Pg 267

https://www.drishtiias.com/loksabha-rajyasabha-discussions/in-depth-biodiversity-warning-bells

https://www.drishtiias.com/loksabha-rajyasabha-discussions/the-big-picture-world-environment-day-identify-the-dangers-of-the-plastic-era

31. Recently, there was a growing awareness in our country about the importance of Himalayan nettle (Girardinia diversifolia) because it is found to be a sustainable source of

(a) anti-malarial drug

(b) biodiesel

(c) pulp for paper industry

(d) textile fibre

Exp: Himalayan nettle(Girardinia diversifolia) is found to be a sustainable source of textile fiber. The fabric and the things made from it. For the Himalayan Indian Nettle, Government of india has granted approval for a project titled Development of products from Himalayan Indian nettle to Department of Jute and Fibre Technology, Institute of Jute Technology, University of Kolkata.

Source: http://pib.nic.in/newsite/PrintRelease.aspx?relid=159342

32. For the measurement/estimation of which of the following are satellite images/remote sensing data used?

  1. Chlorophyll content in the vegetation of a specific location
  2. Greenhouse gas emissions from rice paddies of a specific location
  3. Land surface temperatures of a specific location

Select the correct answer using the code given below.

(a) 1 Only

(b) 2 and 3 only

(c) 3 only

(d) 1, 2 and 3

Exp: Statement 1: The Sentinel satellite of the European Space Agency has certain specific spectral characteristics that can be used to measure even the chlorophyll content in the vegetation of a specific location. Correct.

Statement 2: Estimation of Methane Emissions from Rice Paddies in the Mekong Delta Based on Land Surface Dynamics Characterization with Remote Sensing. Correct.

Statement 3: The land surface temperature of a specific location can be measured with the help of satellite images/ remote sensing data. Correct.

Source: https://www.downtoearth.org.in/interviews/science-and-technology/-remote-sensing-is-being-used-to-predict-crop-growing-seasons--56372

Covered in Drishti: https://www.drishtiias.com/daily-updates/daily-news-analysis/isro-to-launch-state-of-the-art-satellites

33. Consider the following States:

  1. Chhattisgarh
  2. Madhya Pradesh
  3. Maharashtra
  4. Odisha

With reference to the States mentioned above, in terms of percentage of forest cover to the total area of State, which one of the following is the correct ascending order?

(a) 2-3-1-4

(b) 2-3-4-1

(c) 3-2-4-1

(d) 3-2-1-4

Exp: India state of forest report 2017

State Percentage of forest cover to the total area (Approximate data)
1. Chattisgarh 41
2. MP 25
3. Maharashtra 16
4. Orissa 32

Hence, correct ascending order- (3-2-4-1)

Source: http://fsi.nic.in/isfr2017/isfr-forest-cover-2017.pdf

34. Which of the following statements are correct about the deposits of ‘methane hydrate’?

  1. Global warming might trigger the release of methane gas from these deposits.
  2. Large deposits of ‘methane hydrate’ are found in Arctic Tundra and under the seafloor.
  3. Methane in atmosphere oxidizes to carbon dioxide after a decade or two.

Select the correct answer using the code given below.

(a) 1 and 2 only

(b) 2 and 3 only

(c) 1 and 3 only

(d) 1, 2 and 3

Exp: Statement 1 is correct - Methane escaping from disturbed undersea hydrates may be an ecological threat. Even if a small portion of these methane hydrate deposits decompose through natural processes, large quantities of methane will be released to exacerbate global warming.

Statement 2 is correct - Methane hydrates tend to form along the lower margins of continental slopes, where the seabed drops from the relatively shallow shelf, usually to about 150 metres below the sea surface. The susceptibility of gas hydrates to warming climate depends on the duration of the warming event, their depth beneath the seafloor or tundra surface, and the amount of warming required to heat sediments to the point of dissociating gas hydrates.

Statement 3 is correct - The problem with methane is that it does not vanish without a trace, even though it remains in the atmosphere relatively briefly - 10 years on an average. In the presence of free oxygen, a methane molecules single atom of carbon disengages from its four hydrogen atoms to become carbon dioxide.

Source: https://www.downtoearth.org.in/news/burning-ice-17856

Covered in Drishti: https://www.drishtiias.com/daily-updates/daily-news-analysis/iit-madras-team-produces-gas-hydrates

https://www.drishtiias.com/daily-updates/daily-news-analysis/earth-at-risk-of-becoming-hothouse

35. Consider the following:

  1. Carbon monoxide
  2. Methane
  3. Ozone
  4. Sulphur dioxide

Which of the above are released into atmosphere due to the burning of crop/biomass residue?

(a) 1 and 2 only

(b) 2, 3 and 4 only

(c) 1 and 4 only

(d) 1, 2, 3 and 4

Exp: Explanation-Burning of crop residues emits traces of carbon dioxide, methane, carbon monoxide, nitrous oxide, sulphur dioxide and particulates which affect human health.

Punjab and Haryana are both responsible for causing air pollution due to burning. These two states contribute to 48 per cent of the total emission due to paddy burning across India.

Note: The majority of tropospheric ozone formation occurs when nitrogen oxides (NOx), carbon monoxide (CO) and volatile organic compounds (VOCs), such as xylene, react in the atmosphere in the presence of sunlight.

Source: http://www.isca.in/EARTH_SCI/Archive/v1/i1/4.ISCA-IRJES-2013-005.pdf

https://www.downtoearth.org.in/news/air/paddy-burning-ngt-orders-fine-imposition-on-erring-farmers-51698

Covered in Drishti: https://www.drishtiias.com/daily-updates/daily-news-analysis/crop-stubble-burning-challenge-in-punjab

36. Consider the following pairs:

Sea Bordering Country
1. Adriatic Sea Albania
2. Black Sea Croatia
3. Caspian Sea Kazakhstan
4. Mediterranean Sea Morocco
5. Red Sea Syria

Which of the pairs given above are correctly matched?

(a) 1, 2 and 4 only

(b) 1, 3 and 4 only

(c) 2 and 5 only

(d) 1, 2, 3, 4 and 5

Exp: The Adriatic Sea is a part of the Mediterranean Sea positioned between the eastern coastline of Italy, and countries of the Balkan Peninsula, from Slovenia, South through Croatia, Montenegro, and to Albania. Hence, pair 1 is correctly matched.

The Black Sea is an inland sea located between far-southeastern Europe and the far-western edges of the continent of Asia and the country of Turkey. It's bordered by Turkey, and by Bulgaria, Romania, Ukraine, Russia and Georgia. Hence, pair 2 is not correctly matched. this rules out options a, c and d, leaving the correct answer to be option B through elimination method.

Caspian Sea is an enclosed body of water between Asia and Europe. It is bordered by Iran, Turkmenistan, Kazakhstan, Azerbaijan, Russia,

Morocco’s Mediterranean coast represents the westernmost edge of the Northern African coast. The coastline features the Strait of Gibraltar that marks the link between the Mediterranean Sea and the Atlantic Ocean.

There are six countries (Saudi Arabia, Yemen, Egypt, Sudan, Eritrea, and Djibouti) bordering the Red Sea.

Source: https://www.worldatlas.com/

Covered in Drishti: https://www.drishtiias.com/to-the-points/Paper2/india-central-asia-relations

37. Among the following, which one is the largest exporter of rice in the world in the last five years?

(a) China

(b) India

(c) Myanmar

(d) Vietnam

Exp: India has been the world’s top rice exporter since the beginning of this decade. India emerged the world’s largest rice exporter in 2011-12, displacing Thailand from its leadership position. 

Source: India Today, The Hindu, Economic Times, Worldatlas.com

38. Consider the following pairs

Glacier River
1. Bandarpunch Yamuna
2. Bara Shigri Chenab
3. Milam Mandakini
4. Siachen Nubra
5. Zemu Manas

Which of the pairs correctly matched?

(a) 1, 2 and 4

(b) 1, 3 and 4

(c) 2 and 5

(d) 3 and 5

Exp: An important glacier of Yamuna river basin is Bandarpunch Glacier in the Garhwal division of the Himalayas. It is 12 km long situated on the Northern slopes of Bandarpunch West, Khatling peak and Bandarpunch peak the glacier is formed by three cirque glaciers and then join the river of Yamuna. Hence, 1 is correctly matched

Bara Shigri is the largest glacier located in Lahaul Spiti region in Chandra Valley, Himachal Pradesh. It is a 30-km long glacier, the second longest glacier in the Himalayas after Gangotri. It flows northwards and feeds the Chenab river. Hence, Statement 2 is correct.

Milam Glacier in Munsiyari, Pithoragarh district, Uttarakhand is the source of Gori Ganga River and not of Mandakini River. Gori Ganga is also an important tributary of Kali River. Hence, Statement 3 is not correct.

At an altitude of roughly 5,400 meters (17,700 feet), the Siachen Glacier in Kashmir is a forbidding place. At much lower altitudes, the glacier’s impact is benign: it is the source of the Nubra River, a tributary of the Indus River flowing into Pakistan and the Arabian Sea. Hence, Statement 4 is correct.

The Zemu glacier is the largest in the Eastern Himalayas in Sikkim. It is at the base of the Kanchendzonga and is one of the sources for the Teesta not Manas river. Teesta is a tributary of the Brahmaputra.Hence, Statement 5 is not correct.

Source: http://www.visituttrakhand.com/bandarpunch-glacier-trek.php

39. In India, the use of carbofuran, methyl parathion, phorate and triazophos is viewed with apprehension. These chemicals are used as

(a) pesticides in agriculture

(b) preservatives in processed foods

(c) fruit-ripening agents

(d) moisturising agents in cosmetics

Exp: Carbofuran, Phorate and Triazophos are pesticides used in agriculture.

To promote organic farming in Kerala, the state agriculture department had ordered a ban on the use of pesticides on May 7. Kerala Agriculture University was asked to provide alternatives to the banned pesticides, which include carbofuran, phorate, methyl parathion, monocrotophos, methyl demethon, prophenophos and triazophos. The university suggested less hazardous pesticides, like acephate, carbaryl, dimethoate and flubendiamide. Hence option A is correct.

Source: https://www.downtoearth.org.in/news/pesticide-ban-lands-kerala-in-court-33657

Covered in Drishti: https://www.drishtiias.com/daily-updates/daily-news-analysis/pesticide-ban

40. Consider the following statements:

  1. Under Ramsar Convention, it is mandatory on the part of the Government of India to protect and conserve all the wetlands in the territory of India.
  2. The Wetlands (Conservation and Management) Rules, 2010 were framed by the Government of India based on the recommendations of Ramsar Convention.
  3. The Wetlands (Conservation and Management) Rules, 2010 also encompass the drainage area or catchment regions of the wetlands as determined by the authority.

Which of the statements given above is/are correct?

(a) 1 and 2 only

(b) 2 and 3 only

(c) 3 only

(d) 1, 2 and 3

Exp: Statement 1 is Not correct- Under Ramsar Convention it is mandatory on the part of government of India to protect only Ramsar Sites and not all the wetlands in India.

Statement 3 is Correct- Under the Wetlands (Conservation and Management Rules), 2010, Wetland" means an area or of marsh, fen, peatland or water; natural or artificial, permanent or temporary, with water that is static or flowing, fresh, brackish or salt, including areas of marine water, the depth of which at low tide does not exceed six metres and includes all inland waters such as lakes, reservoir, tanks, backwaters, lagoon, creeks, estuaries and man made wetland and the zone of direct influence on wetlands that is to say the drainage area or catchment region of the wetlands as determined by the authority but does not include main river channels, paddy fields and the coastal wetland.

Due to confusion on veracity of statement 2, we are yet to arrive at definite a conclusion.

Source: https://www.downtoearth.org.in/coverage/towards-wetland-conservation-33192

https://indiankanoon.org/doc/106740276/

Covered in Drishti: https://www.drishtiias.com/daily-updates/daily-news-analysis/world-wetlands-day

41. Consider the following statements:

  1. Agricultural soils release nitrogen oxides into environment.
  2. Cattle release ammonia into environment.
  3. Poultry industry releases reactive nitrogen compounds environment.

Which of the statements given above is/arc correct?

(a) 1 and 3 only

(b) 2 and 3 only

(c) 2 only

(d) 1, 2 and 3

Exp: Statement 1 is correct: Agricultural soils contributed to over 70% of N2O emissions from India in 2010, followed by waste water (12%) and residential and commercial activities (6%).

Statement 2 is correct: Cattle account for 80% of the ammonia production and India is globally the biggest source of ammonia emission, nearly double that of NOx emissions.

Statement 3 is correct: The poultry industry, with an annual growth rate of 6%, recorded an excretion of reactive nitrogen compounds of 0.415 tonnes in 2016.

Source: https://www.thehindu.com/sci-tech/energy-and-environment/nitrogen-emissions-going-up-study/article24090131.ece

42. What is common to the places known as Aliyar, Isapur and Kangsabati?

(a) Recently discovered uranium deposits

(b) Tropical rain forests

(c) Underground cave systems

(d) Water reservoirs

Exp: Aliyar (Tamil Nadu) Isapur (Maharashtra) and Kangsabati (West Bengal) are water reservoirs, where water level reached much below the normal capacity.

Source: https://www.downtoearth.org.in/news/water/water-level-in-91-reservoirs-at-36-per-cent-of-total-capacity-southern-states-most-affected-59826

43. In the context of proposals to the use of hydrogen-enriched CNG (H-CNG) as fuel for buses in public transport, consider the following statements:

  1. The main advantage of the use of H-CNG is the elimination of carbon monoxide emissions.
  2. H-CNG as fuel reduces carbon dioxide and hydrocarbon emissions.
  3. Hydrogen up to one-fifth by volume can be blended with CNG as fuel for buses.
  4. H-CNG makes the fuel less expensive than CNG.

Which of the statements given above is/are correct?

(a) 1 only

(b) 2 and 3 only

(c) 4 only

(d) 1, 2, 3 and 4

Exp: Hydrogen-Enriched CNG (H-CNG) is a blend of hydrogen and CNG, the ideal hydrogen concentration being 18%. Hence, Statement 3 is correct.

Compared to conventional CNG, use of H-CNG can reduce emission of carbon monoxide up to 70%,not complete emission. Hence, statement 1 is not correct. It also reduces carbondioxide and hydrocarbon emissions. Hence, statement 2 is correct.

H-CNG also enables up to 5% savings in fuel efficiency. H- CNG also reduces the engines unburned hydrocarbon emissions and speed up the process of combustion.

Source: https://indianexpress.com/article/explained/cng-to-hydrogen-cng-why-switch-and-how-5278356/

44. Why are dewdrops not formed on a cloudy night?

(a) Clouds absorb the radiation released from the Earth’s surface.

(b) Clouds reflect back the Earth’s radiation.

(c) The Earth’s surface would have low temperature on cloudy nights.

(d) Clouds deflect the blowing wind to ground level.

Exp: Dew is a type of precipitation where water droplets form on the ground, or on objects near the ground in a process called condensation of moisture. The favorable weather elements for dew include clear skies, light wind, decent soil moisture, and low night-time dew point depressions.

Dew forms when the temperature becomes equal to the dew point. This often happens first at ground level for two reasons. First, longwave emission causes the earth's surface to cool at night.Cloud cover prevent this from happening as it reflect's earth's radiation.

Condensation requires the temperature to decrease to the dew point. Second, the soil is often the moisture source for the dew. Warm and moist soils will help with the formation of dew as the soil cools overnight.

Source: https://www.weather.gov/source/zhu/ZHU_Training_Page/fog_stuff/Dew_Frost/Dew_Frost.htm

45. Consider the following statements:

  1. The 44th Amendment to the Constitution of India introduced an Article placing the election of the Prime Minister beyond judicial review.
  2. The Supreme Court of India struck down the 99th Amendment to the Constitution of India as being violative of the independence of judiciary.

Which of the statements given above is/are correct?

(a) 1 only

(b) 2 only

(c) Both 1 and 2

(d) Neither 1 nor 2

Exp: In 1975, Parliament passed the Thirty-ninth amendment to the Constitution which removed the authority of the Supreme Court to adjudicate petitions regarding elections of the President, Vice President, Prime Minister and Speaker of the Lok Sabha. Instead, a body constituted by Parliament would be vested with the power to resolve such election disputes.

So, option 1 is incorrect.

Supreme court struck down 99th amendment as it impinges upon the principles of “independence of the judiciary”, as well as, “separation of powers”

Source: https://www.sci.gov.in/pdf/aorexam/List%20of%20leading%20cases%20and%20short%20notes%20given%20by%20the%20Mr.%20F.S.%20Nariman,%20Sr%20Adv%20&%20Chairman,%20Bd.%20of%20Examiner%20of%20the%20AOR%20Exam%202016.pdf

46. Consider the following statements:

  1. The motion to impeach a Judge of the Supreme Court of India cannot be rejected by the Speaker of the Lok Sabha as per the Judges (Inquiry) Act 1968.
  2. The Constitution of India defines and gives details or what constitutes ‘incapacity and proved misbehaviour’ of the Judges of the Supreme Court of India.
  3. The details of the process of impeachment of the Judges of the Supreme Court of India are given in 4 the Judges (Inquiry) Act, 1968.
  4. If the motion for the impeachment of a Judge is taken up for voting, the law requires the motion to be backed by each House of the Parliament and supported by a majority of total membership of that House and by not less than two-thirds of total members of that House present and voting.

Which of the statements given above is/are correct?

(a) 1 and 2

(b) 3 only

(c) 3 and 4 only

(d) 1, 3 and 4

Exp: The Judges Enquiry Act (1968) regulates the procedure relating to the removal of a judge of the Supreme Court by the process of impeachment. Hence, statement 3 is correct.

  1. A removal motion signed by 100 members (in the case of Lok Sabha) or 50 members (in the case of Rajya Sabha) is to be given to the Speaker/Chairman.
  2. The Speaker/Chairman may admit the motion or refuse to admit it. Hence statement 1 is not correct.
  3. If it is admitted, then the Speaker/Chairman is to constitute a three-member committee to investigate into the charges.
  4. The committee should consist of (a) the chief justice or a judge of the Supreme Court, (b) a chief justice of a high court, and (c) a distinguished jurist.
  5. If the committee finds the judge to be guilty of misbehaviour or suffering from an incapacity, the House can take up the consideration of the motion.
  6. After the motion is passed by each House of Parliament by special majority, an address is presented to the president for removal of the judge. Hence, Statement 4 is correct. Statement 1 is incorrect and 4 is correct by elimination we can see that only option C is correct.
  7. Finally, the president passes an order removing the judge.

Statement 2 is not correct: The constitution nowhere defines the term "incapacity and proved misbehaviour".

Source: https://www.prsindia.org/theprsblog/explainer-removal-judges-office

47. The Ninth Schedule was introduced in the Constitution of India during the prime ministership of

(a) Jawaharlal Nehru

(b) Lal Bahadur Shastri

(c) Indira Gandhi

(d) Morarji Desai

Exp: In the aftermath of Kameshwar Singh V State of Bihar judgment, the Government got apprehensive that the whole agrarian reform programmes would be endangered. To ensure that agrarian reform legislation did not run into heavy weather, the legislature amended the Constitution in the year 1951 under the Prime ministership of Jawahar Lal Nehru which inserted Ninth Schedule.

Source: https://www.prsindia.org/uploads/media/Judges%20(Inquiry)%20/bill88_2007100588_The_Judges__Inquiry__Act_1968.pdf

48. Consider the following statements:

  1. Coal sector was nationalized by the Government of India under Indira Gandhi.
  2. Now, coal blocks are allocated on lottery basis.
  3. Till recently, India imported coal to meet the shortages of domestic supply, but now India is self-sufficient in coal production.

Which of the statements given above is/are correct?

(a) 1 only

(b) 2 and 3 only

(c) 3 only

(d) 1, 2 and 3

Exp: i. Coal sector was nationalised in two phases under Indira Gandhi Government in 1972.

ii. The coal Blocks are allocated through Auctions not on lottery basis.

iii. The coal sector is the monopolistic sector in India. India holds 5th biggest coal reserves in the world but due to incapacity of coal production by monopolistic firms, it imports coals to meet the shortages of domestic supply. But, still the country is not self-sufficient of coal production.

Hence option (A) is correct

Source: https://coal.nic.in/content/historybackground

https://blog.ipleaders.in/coal-blocks/

49. Consider the following statements:

  1. The Parliament (Prevention of Disqualification) Act, 1959 exempts several posts from disqualification on the grounds of ‘Office of Profit’.
  2. The above-mentioned Act was amended five times.
  3. The term ‘Office of Profit’ is well-defined in the Constitution of India.

Which of the statements given above is/are correct?

(a) 1 and 2 only

(b) 3 only

(c) 2 and 3 only

(d) 1, 2 and 3

Exp: Statement 1 - The Parliament (Prevention of Disqualification) Act, 1959 exempts several posts from disqualification. Like–

(i) Ministers of State and Deputy Ministers,

(ii) Parliamentary Secretaries and Parliamentary Under Secretaries,

(iii) Deputy Chief Whips in Parliament,

(iv) Vice-Chancellors of Universities,

(v) Officers in the National Cadet Corps, and the Territorial Army, and

(vi) the Chairman and members of Advisory Committees set up by the Government when they are not entitled to any fee or remuneration other than compensatory Etc. Hence, Statement 1 is correct.

Statement 2 It has been amended 5 times since its formulation 1960, 19992,1993,2006 and 2013. Hence, statement 2 is correct.

Statement 3 The law or the constitution of India does not clearly define what constitutes an office of profit but the definition has evolved over the years with interpretations made in various court judgments. Hence Statement 3 is incorrect. By this information we can eliminate option b, c and d.

50. Under which Schedule of the Constitution of India can the transfer of tribal land to private parties for mining be declared null and void?

(a) Third Schedule

(b) Fifth Schedule

(c) Ninth Schedule

(d) Twelfth Schedule

Exp: Grant of mineral concessions in Scheduled Areas is guided by the provisions contained in article 244 along with Fifth and Sixth Schedules to the Constitution relating to administration of the Scheduled Areas and Tribal Areas and the provisions of the Panchayats (Extension to the Scheduled Areas) Act, 1996 and the Scheduled Tribes and Other Traditional Forest Dwellers (Recognition of Forest Rights) Act, 2006 or any other relevant statutory acts protecting the interests of tribals.

Under the fifth Schedule, the Governor may by public notification direct that any particular Act of Parliament or of the Legislature of the State shall not apply to a Scheduled Area or any part thereof in the State or shall apply to a Scheduled Area.

Source: https://mines.gov.in/writereaddata/UploadFile/NMP12032019.pdf

51. Consider the following statements about Particularly Vulnerable Tribal Groups (PVTGs) in India:

  1. PVTGs reside in 18 States and one Union Territory.
  2. A stagnant or declining population is one of the criteria for determining PVTG status.
  3. There are 95 PVTGs officially notified in the country so far.
  4. Irular and Konda Reddi tribes are included in the list of PVTGs.

Which of the statements given above are correct?

(a) 1, 2 and 3

(b) 2, 3 and 4

(c) 1, 2 and 4

(d) 1, 3 and 4

Exp: There are 75 Particularly Vulnerable Tribal Groups (PVTGs), not 95, notified as on date in the country. The criteria followed for determination of PVTGs are as under:

i. A pre-agriculture level of technology;

ii. A stagnant or declining population;

iii. Extremely low literacy; and

iv. A subsistence level of the economy. Hence, statement 3 is incorrect, which gives us option C as the correct answer by elimination method.

Source: http://pib.nic.in/newsite/PrintRelease.aspx?relid=178257

https://tribal.nic.in/DivisionsFiles/SwLPVTGs.pdf

Covered in Drishti: https://www.drishtiias.com/summary-of-important-reports/xaxa-committee-on-tribal-communities-of-india

52. With reference to the Constitution of India, prohibitions or limitations or provisions contained in ordinary laws cannot act as prohibitions or limitations on the constitutional powers under Article 142. It could mean which one of the following?

(a) The decisions taken by the. Election Commission of India while discharging its duties cannot be challenged in any court of law.

(b) The Supreme Court of India is not constrained in the exercise of its powers by the laws made by Parliament.

(c) In the event of grave financial crisis in the country, the President of India can declare Financial Emergency without the counsel from the Cabinet.

(d) State Legislatures cannot make laws on certain matters without the concurrence of Union Legislature.

Exp: Article142- Enforcement of decrees and orders of Supreme Court and unless as to discovery, etc (1) The Supreme Court in the exercise of its jurisdiction may pass such decree or make such order as is necessary for doing complete justice in any cause or matter pending before it, and any decree so passed or orders so made shall be enforceable throughout the territory of India in such manner as may be prescribed by or under any law made by Parliament and, until provision in that behalf is so made, in such manner as the President may by order prescribe.

Source: https://indiankanoon.org/doc/500307/

Covered in Drishti: https://www.drishtiias.com/to-the-points/Paper2/witness-protection-scheme-2018-to-the-point

53. With reference to the Legislative Assembly of a State in India, consider the following statements:

  1. The Governor makes a customary address to Members of the House at the commencement of the first session of the year.
  2. When a State Legislature does not have a rule on a particular matter, it follows the Lok Sabha rule on that matter.

Which of the statements given above is/are correct?

(a) 1 only

(b) 2 only

(c) Both 1 and 2

(d) Neither 1 nor 2

Exp: Statement 1 is correct: Article 176(1) of the Constitution of India enjoins that the Governor shall Address both the Houses assembled together at the commencement of the first Session after each general election to the Assembly and at the commencement of the first session of each year and inform the Legislature of the causes of its Summons.

Statement 2 is incorrect: In case of absence of rules in particular matter, the speaker of state legislative assembly/council decides rules.

208. Rules of procedure

(1) A House of the Legislature of a State may make rules for regulating subject to the provisions of this Constitution, its procedure and the conduct of its business

(2) Until rules are made under clause (1), the rules of procedure and standing orders in force immediately before the commencement of this Constitution with respect to the Legislature for the corresponding Province shall have effect in relation to the Legislature of the State subject to such modifications and adaptations as may be made therein by the Speaker of the Legislative Assembly, or the Chairman of the Legislative Council, as the case may be

(3) In a State having a Legislative Council the Governor, after consultation with the Speaker of the Legislative Assembly and the Chairman of the Legislative Council, may make rules as to the procedure with respect to communications between the two Houses.

Source: https://indiankanoon.org/doc/695063/

54. Consider the following statements:

  1. The United Nations Convention against Corruption (UNCAC) has a ‘Protocol against the Smuggling of Migrants by Land, Sea and Air’.
  2. The UNCAC is the ever-first legally binding global anti-corruption instrument.
  3. A highlight of the United Nations Convention against Transnational Organized Crime (UNTOC) is the inclusion of a specific chapter aimed at returning assets to their rightful owners from whom they had been taken illicitly.
  4. The United Nations Office on Drugs and Crime (UNODC) is mandated by its member States to assist in the implementation of both UNCAC and UNTOC.

Which of the statements given above are correct?

(a) 1 and 3 only

(b) 2, 3 and 4 only

(c) 2 and 4 only

(d) 1, 2, 3 and 4

Exp:  (i) The protocol against the smuggling of Migrants by land, sea and air, it comes under the the United Nations Convention against Transnational Organized Crime (UNTOC). The main objective of this conventon is to fight against Transnational organized crime. Hence statement 1 is incorrect.

(ii) The United Nations Convention Against Corruption (UNCAC) is the only legally binding universal anti-corruption instrument. The Convention covers many different forms of corruption, such as bribery, trading in influence, abuse of functions, and various acts of corruption in the private sector. Hence statement 2 is correct.

(iii) "Returning Assets to their rightful owners from whom they they have been taken illicitly", is included under the United Nations Convention Against Corruption (UNCAC). Hence statement 3 is incorrect. Eliminating options containing statement 3 also gives us the answer, that is option C.

(iv) The United Nations Office on Drugs and Crime (UNODC) is mandated by its Member States to assist in the implementation of both Conventions,such as UNCAC and UNTOC, along with the UN Drug Conventions of 1961, 1971 and 1988 underpin all the operational work of UNODC. Hence statement 4 is correct.

Therefore option (c) is correct.

Source: United Nations Website

Covered in Drishti: https://www.drishtiias.com/important-institutions/drishti-specials-important-institutions-international-institution/united-nations-1

55. Consider the following statements:

  1. As per recent amendment to the Indian Forest Act, 1927, forest dwellers have the right to fell the bamboos grown on forest areas.
  2. As per the Scheduled Tribes and Other Traditional Forest Dwellers (Recognition of Forest Rights) Act, 2006, bamboo is a minor forest produce.
  3. The Scheduled Tribes and Other Traditional Forest Dwellers (Recognition of Forest Rights) Act, 2006 allows ownership of minor forest produce to forest dwellers.

Which of the statements given above is/are correct?

(a) 1 and 2 only

(b) 2 and 3 only

(c) 3 only

(d) 1, 2 and 3

Exp: The Indian Forest (Amendment) Bill 2018 permits felling and transit of bamboo grown in non-forest areas. However, bamboo grown on forest lands would continue to be classified as a tree and would be guided by the existing legal restrictions. Hence, staement 1 is not correct. The Forest Rights Act (FRA) of 2006 recognises bamboo as an Minor Forest Produce and vests the “right of ownership, access to collect, use and dispose of minor forest pr oduce” with Scheduled Tribes and traditional forest dwellers. Hence statement 2 is correct. In 2006, the Forest Rights Act (FRA) for the first time defined minor forest produce as including bamboo and tendu and many other things. It also gave tribals and other traditional forest dwellers the “right of ownership, access to collect, use and dispose of minor forest produce, which has been traditionally collected within or outside village boundaries.” Hence statement 3 is correct

Source: https://www.thehindu.com/news/national/bamboo-not-a-tree-parliament-passes-bill-amending-forest-act/article22287886.ece

56. Which Article of the Constitution of India safeguards one’s right to marry the person of one’s choice?

(a) Article 19

(b) Article 21

(c) Article 25

(d) Article 29

Exp: The right to marry is a component of right to life under art 21 of constitution of India which says, “No person shall be deprived of his life and personal liberty except according to procedure established by law”. in the context of right to marry, mention may be made of a few Indian cases. Person who suffering from venereal disease, even prior to the marriage cannot be said to have any right to marry so long as he is not fully cured of the disease.

In Lata singh vs state of Uttar Pradesh 2006, the Supreme Court viewed the right to marry as a component of right to life under Art 21 of Indian Constitution the court observed that:

“This is a free and democratic country, and once a person becomes a major he or she can marry whosoever he/she likes. If the parents of the boy or girl do not approve of such inter-caste marriage the maximum they can do is that they can cut off social relations with the son or daughter, but they cannot give threats or commit or instigate acts of violence and cannot harass the person who undergoes such intercaste marriage”.

Source: http://www.legalservicesindia.com/article/1001/Right-to-marry.html

57. Consider the following statements:

  1. According to the Indian Patents Act, a biological process to create a seed can be patented in India.
  2. In India, there is no Intellectual Property Appellate Board.
  3. Plant varieties are not eligible to be patented in India.

Which of the statements given above is/are correct?

(a) 1 and 3 only

(b) 2 and 3 only

(c) 3 only

(d) 1, 2 and 3

Exp: Article 3(J) of Indian Patent Act, excludes from patentability “plants and animals in whole or in any part thereof other than microorganisms, including seeds, varieties, and species, and essentially biological processes for production or propagation of plants and animals”. Hence statement 1 is not correct.

The Intellectual Property Appellate Board (IPAB) was constituted on 2003 by the Government of India to hear and resolve the appeals against the decisions of the registrar under the Indian Trademarks Act, 1999 and the Geographical Indications of Goods (Registration and Protection) Act, 1999. Hence statement 2 is incorrect

Plant variety protection provides legal protection of a plant variety to a breeder in the form of Plant Breeder’s Rights (PBRs). In India, the Plant Variety Protection And Farmers Rights (PPVFR) Act, 2001 is a sui generis system that aims to provide for the establishment of an effective system for the protection of plant varieties and the rights of plant breeders and farmers. Hence statement 3 is correct.

Hence option C is correct.

58. Consider the following statements:

The Environment Protection Act, 1986 empowers the Government of India to

  1. state the requirement of public participation in the process of environmental protection, and the procedure and manner in which it is sought
  2. lay down the standards for emission or discharge of environmental pollutants from various sources

Which of the statements given above is/are correct?

(a) 1 only

(b) 2 only

(c) Both 1 and 2

(d) Neither 1 nor 2

Exp: Statement 2 is correct- The Environment Protection Act 1986, empowers the government of India to lay down standards for emission or discharge of environmental pollutants from various sources whatsoever, provided that different standards for emission or discharge may be laid down under this clause from different sources having regard to the quality or composition of the emission or discharge of environmental pollutants from such sources.

Source: https://indiacode.nic.in/bitstream/123456789/4316/1/ep_act_1986.pdf

59. As per the Solid Waste Management Rules, 2016 in India, which one of the following statements is correct?

(a) Waste generator has to segregate waste into five categories.

(b) The Rules are applicable to notified urban local bodies, notified towns and all industrial townships only.

(c) The Rules provide for exact and elaborate criteria for the identification of sites for landfills and waste processing facilities.

(d) It is mandatory on the part of waste generator that the waste generated in one district cannot be moved to another district.

Exp: Under the Solid Waste Management, Rules 2016, waste processing facilities will have to be set up by all local bodies having 1 million or more population within two years. In case of census towns below 1 million population, setting up common, or stand-alone sanitary landfills by, or for all local bodies having 0.5 million or more population and for setting up common, or regional sanitary landfills by all local bodies and census towns under 0.5 million population will have to be completed in three years. Hence, rules provide for exact and elaborate criteria for identification of landfill sites and waste processing facilities. Hence option C is correct.

Waste generators would now have to now segregate waste into three streams- Biodegradables, Dry (Plastic, Paper, metal, Wood, etc.) and Domestic Hazardous waste (diapers, napkins, mosquito repellants, cleaning agents etc.) before handing it over to the collector. Hence, option a is incorrect.

Source: http://pib.nic.in/newsite/PrintRelease.aspx?relid=138591 

60. Consider the following statements:

As per the Industrial Employment (Standing Orders) Central (Amendment) Rules, 2018

  1. if rules for fixed-term employment are implemented, it becomes easier for the firms/companies to lay off workers
  2. no notice of termination of employment shall be necessary in the case of temporary workman

Which of the statements given above is/are correct?

(a) 1 only

(b) 2 only

(c) Both 1 and 2

(d) Neither 1 nor 2

Exp: The government has notified fixed term employment for all sectors through an amendment to the Industrial Employment (Standing Orders) Central Rules, 1946. Fixed-term employment for all sectors will make it easier for companies to hire-and-fire workers along with reducing the role of middlemen. Hence, statement 1 is correct. No notice of termination of employment shall be necessary in the case of temporary workman whether monthly rated, weekly rated or piece rated and probationers or badli workmen. Hence, statement 2 is correct.

Source: https://labour.gov.in/sites/default/files/FTE%20Final%20Notification.pdf

61. The Service Area Approach was implemented under the purview of

(a) Integrated Rural Development Programme

(b) Lead Bank Scheme

(c) Mahatma Gandhi National Rural Employment Guarantee Scheme

(d) National Skill Development Mission

Exp: Under the Lead bank Scheme, the service area approach was introduced in 1989 for planned and orderly development of rural and semi-urban areas. Each bank branch in rural and semi-urban area was designated to serve an area if 15 to 25 villages and the branch was responsible for meeting the needs of bank credit of its service area.

Source: RBI Website https://rbi.org.in/scripts/BS_ViewMasCirculardetails.aspx?id=9853

Covered in Drishti: DLP Book for UPPCS Mains/Indian Economy-I/Chapter: Bankng & Monetary Policy

62. With reference to the management of minor minerals in India, consider the following statements:

  1. Sand is a ‘minor mineral’ according to the prevailing law in the country.
  2. State Governments have the power to grant mining leases of minor minerals, but the powers regarding the formation of rules related to the grant of minor minerals lie with the Central Government.
  3. Stale Governments have the power to frame rules to prevent illegal mining of minor minerals.

Which of the statements given above is/are correct?

(a) 1 and 3 only

(b) 2 and 3 only

(c) 3 only

(d) 1, 2 and 3

Exp: Sand is a minor mineral, as defined under section 3(e) of the Mines and Minerals (Development and Regulation) Act, 1957 (MMDR Act). Hence statement 1 is correct. Section 15 of the MMDR Act empowers state governments to make rules for regulating the grant of mineral concessions in respect of minor minerals and for purposes connected therewith. The regulation of grant of mineral concessions for minor minerals is, therefore, within the legislative and administrative domain of the state governments. Under the power granted to them by section 15 of the MMDR Act, State Governments have framed their own minor minerals concession rules. Hence statement 2 in incorrect. section 23C of theMMDR Act, 1957 empowers state governments to frame rules to prevent illegal mining, transportation and storage of mineralsand for purposes connected therewith. Control of illegal mining is, therefore, under the legislative and administrative jurisdiction of state governments. Hence statement 3 is correct.

Source: http://pib.nic.in/newsite/PrintRelease.aspx?relid=155423

63. Consider the following statements:

  1. Most of India’s external debt is owed by governmental entities.
  2. All of India’s external debt is denominated in US dollars.

Which of the statements given above is/are correct?

(a) 1 only

(b) 2 only

(c) Both 1 and 2

(d) Neither 1 nor 2

Exp: US dollar denominated debt is the largest component of India's External Debt, followed by Indian rupee,SDR,Yen and Euro. Hence statement 2 is not correct. Commercial Borrowings continue to be the largest componenet of India's external Debt followed by NRI deposit and short term credit. Hence statement 1 is not correct.

Source: https://www.business-standard.com/article/news-cm/india-s-external-debt-eases-to-us-521-2-billion-end-december-2018-119040100925_1.html

64. Which of the following is not included in the assets of a commercial bank in India?

(a) Advances

(b) Deposits

(c) Investments

(d) Money at call and short notice

Exp: The deposit itself is a liability owed by the bank to the depositor. Bank deposits refer to this liability rather than to the actual funds that have been deposit. Hence option B is the correct answer.

Source: https://www.investopedia.com/terms/b/bank-deposits.asp

65. In the context of India, which of the following factors is/are contributor/contributors to reducing the risk of a currency crisis?

  1. The foreign currency earnings of India’s IT sector
  2. Increasing the government expenditure
  3. Remittances from Indians abroad

Select the correct answer using the code given below:

(a) 1 only

(b) 1 and 3 only

(c) 2 only

(d) 1, 2 and 3

Exp: A currency crisis is brought on by a decline in the value of a country's currency. This decline in value negatively affects an economy by creating instabilities in exchange rates, meaning that one unit of a certain currency no longer buys as much as it used to in another currency.

Foreign currency earnings and Remittances contribute to the strengthening of the rupee. Hence statement 1 and 3 is correct.

Increasing government Expenditure will have no effect on the the value of currency. Hence statement 2 is not correct.

Source: https://www.thehindu.com/business/Economy/rbi-move-on-remittance-to-help-strengthen-rupee/article3528924.ece#!

66. Which one of the following suggested that the Governor should be an eminent person from outside the State and should be a detached figure without intense political links or should not have taken part in politics in the recent past?

(a) First Administrative Reforms Commission (1966)

(b) Rajamannar Committee (1969)

(c) Sarkaria Commission (1983)

(d) National Commission to Review the Working of the Constitution (2000)

Exp: Sarkaria commission gave its recommendations for the selection of Governors. 

Source: http://interstatecouncil.nic.in/wp-content/uploads/2015/06/CHAPTERIV.pdf

Covered in Drishti: https://www.drishtiias.com/to-the-points/Paper2/governor

67. Which of the following is issued by registered foreign portfolio investors to overseas investors who want to be part of the Indian stock market without registering themselves directly?

(a) Certificate of Deposit

(b) Commercial Paper

(c) Promissory Note

(d) Participatory Note

Exp: P-Notes or Participatory Notes are Overseas Derivative Instruments that have Indian stocks as their underlying assets. They allow foreign investors to buy stocks listed on Indian exchanges without being registered.

Source: https://www.investopedia.com/terms/p/participatorynotes.asp

Covered in Drishti: https://www.drishtiias.com/daily-updates/daily-news-analysis/investments-through-p-notes

68. Consider the following statements:

  1. As per law, the Compensatory Afforestation Fund Management and Planning Authority exists at both National and State levels.
  2. People’s participation is mandatory in the compensatory afforestation programmes carried out under the Compensatory Afforestation Fund Act, 2016.

Which of the statements given above is/are correct?

(a) 1 only

(b) 2 only

(c) Both 1 and 2

(d) Neither 1 nor 2

Exp: The Compensatory Afforestation Fund Act 2016, establishes the National Compensatory Afforestation Fund under the Public Account of India, and a State Compensatory Afforestation Fund under the Public Account of each state.these funds are to be managed by the National and State Compensatory Afforestation Fund Management and Planning Authorities, also set up under the Act. However, there is no mention in the Act of mandatory people's participation in carrying out programmes under the Act. Hence, option a) is correct. 

69. In India, which of the following review the independent regulators in sectors like telecommunications, insurance, electricity, etc.?

  1. Ad Hoc Committees set up by the Parliament
  2. Parliamentary Department Related Standing Committees
  3. Finance Commission
  4. Financial Sector Legislative Reforms Commission
  5. NITI Aayog

Select the correct answer using the code given below:

(a) 1 and 2

(b) 1, 3 and 4

(c) 3, 4 and 5

(d) 2 and 5

Exp: In India, there are 24 Department Related Standing Committees that comprise members from both Houses of Parliament. These committees are ministry specific, and may review the working of regulators within their respective departments. For example, in August 2012, the Standing Committee on Energy presented a report on the functioning of the Central Electricity Regulatory Commission’. In 2011, the Standing Committee on Information Technology had listed the functioning of TRAI for examination. Parliament may also establish ad-hoc committees which may examine the working of regulators. For instance, the terms of reference of the Joint Parliamentary Committee (JPC) on the allocation of 2G spectrum include the review of the policy on spectrum pricing and grant of telecom licences. Another example of parliamentary oversight through ad-hoc committees is the scrutiny of the working of SEBI and RBI by the JPC on the stock market scam. Hence, option a) is correct. 

Source: https://www.prsindia.org/sites/default/files/parliament_or_policy_pdfs/1370586800_Parliamentary%20Oversight%20of%20Regulators.pdf

70. With reference to India’s Five-Year Plans, which of the following statements is/are correct?

  1. From the Second Five-Year Plan, there was a determined thrust towards substitution of basic and capital good industries.
  2. The Fourth Five-Year Plan adopted the objective of correcting the earlier trend of increased concentration of wealth and economic power.
  3. In the Fifth Five-Year Plan, for the first time, the financial sector was included as an integral part of the Plan.

Select the correct answer using the code given below:

(a) 1 and 2 only

(b) 2 only

(c) 3 only

(d) 1, 2 and 3

Exp: The stated objective of the Second five year plan was rapid industrialisation with particular emphasis on the development of basic and heavy industries - aimed at establishing the Socialistic Pattern of Society.

Fourth five year plan emphasised on reduction of concentration of incomes, wealth and economic power to achieve social equality and justice.

Fifth five year plan aims at removal of poverty and achievement of self-reliance. The Eighth Plan focussed on Financial sector reforms.

Source: http://planningcommission.nic.in/plans/planrel/fiveyr/4th/4ppre.htm

71. With reference to Asian Infrastructure Investment Bank (AIIB), consider the following statements:

  1. AIIB has more than 80 member nations.
  2. India is the largest shareholder in AIIB.
  3. AIIB does not have any members from outside Asia.

Which of the statements given above is/are correct?

(a) 1 only

(b) 2 and 3 only

(c) 1 and 3 only

(d) 1, 2 and 3

Exp: The AIIB is a multilateral development bank have now grown to 97 approved members worldwide. Australia, NZ, Qatar, Canada, Cyprus etc are non regional members. Hence statement 3 is not correct. Source AIIB. India is the second largest shareholder of the AIIB. (business standard) India is the largest borrower from Asian Infrastructure Investment Bank

Source: http://pib.nic.in/newsite/PrintRelease.aspx?relid=189031

https://www.aiib.org/en/about-aiib/governance/members-of-bank/index.html

Covered in Drishti: https://www.drishtiias.com/daily-updates/daily-news-analysis/aiib-approves-455-million-dollar-loan-for-andhra-pradesh

72. What was the purpose of Inter-Creditor Agreement signed by Indian banks and financial institutions recently?

(a) To lessen the Government of India’s perennial burden of fiscal deficit and current account deficit

(b) To support the infrastructure projects of Central and State Governments

(c) To act as independent regulator in case of applications for loans of RS 50 crore or more

(d) To aim at faster resolution of stressed assets of Rs 50 crore or more which are under consortium lending

Exp: The inter-creditor agreement is aimed at the resolution of loan accounts with a size of ₹50 crore and above that are under the control of a group of lenders. It is part of the “Sashakt” plan approved by the government to address the problem of resolving bad loans.The Intercreditor Agreement, a recommendation of the Sunil Mehta committee, aims to deal with bad loansas an attempt to speed up the Resolution of stressed Assets.

Source: https://www.thehindubusinessline.com/companies/announcements/others/indian-bank-inter-creditor-agreement-for-resolution-of-stressed-assets/article24809734.ece

https://www.thehindu.com/opinion/editorial/cosmetic-repair/article24506004.ece

73. The Chairmen of public sector banks are selected by the

(a) Banks Board Bureau

(b) Reserve Bank of India

(c) Union Ministry of Finance

(d) Management of concerned bank

Exp: Bank Board Bureau is responsible for selection and appointment of Board of Directors in Public Sector Banks and Financial Institutions.

Source: https://financialservices.gov.in/sites/default/files/Functions%20of%20bank%20board%20bureau.pdf

https://www.livemint.com/

74. Consider the following statements:

  1. Petroleum and Natural Gas Regulatory Board (PNGRB) is the first regulatory body set up by the Government of India.
  2. One of the tasks of PNGRB is to ensure competitive markets for gas.
  3. Appeals against the decisions of PNGRB go before the Appellate Tribunals for Electricity.

Which of the statements given above are correct?

(a) 1 and 2 only

(b) 2 and 3 only

(c) 1 and 3 only

(d) 1, 2 and 3

Exp: The Petroleum and Natural Gas Regulatory Board (PNGRB) was constituted under The Petroleum and Natural Gas Regulatory Board Act, 2006. The independent regulator TRAI is the first independent regulator in India.

Source: http://www.pngrb.gov.in/about-us.html

https://www.prsindia.org/billtrack/the-petroleum-and-natural-gas-regulatory-board-bill-2005-430

https://www.oecd.org/gov/regulatory-policy/44925979.pdf

75. With reference to communication technologies, what is/are the difference/differences between LTE (Long-Term Evolution) and VoLTE (Voice over Long-Term Evolution)?

  1. LTE is commonly marketed as 3G and VoLTE is commonly marketed as advanced 3G.
  2. LTE is data-only technology and VoLTE is voice-only technology.

Select the correct answer using the code given below.

(a) 1 only

(b) 2 only

(c) Both 1 and 2

(d) Neither 1 nor 2

Exp: LTE improved on the radio technology of 3G’s WCDMA and introduced OFDM or Orthogonal Frequency-Division Multiplexing. ITU (The International Telecommunication Union) recognises the lack of clarity in the term 4G and has determined LTE-Advanced and WirelessMAN-Advanced or WiMax 2 as “True” 4G.

VoLTE is a technology update to the LTE protocol used by mobile phone networks. Under LTE, the infrastructure of telecom players only allows transmission of data while voice calls are routed to their older 2G or 3G networks.

Basically VoLTE systems covert voice into data stream, which is then transmitted using the data connection.

Source: https://www.thehindubusinessline.com/opinion/columns/slate/all-you-wanted-to-know-about-volte/article9864031.ece

http://tec.gov.in/pdf/Studypaper/Study%20Paper%20on%20VoLTE%20-%20Final%20(1).pdf

76. Which of the following statements is/are correct regarding the Maternity Benefit (Amendment) Act, 2017?

  1. Pregnant women are entitled for three months pre-delivery and three months post-delivery paid leave.
  2. Enterprises with creches must allow the mother minimum six creche visits daily.
  3. Women with two children get reduced entitlements.

Select the correct answer using the code given below.

(a) 1 and 2 only

(b) 2 only

(c) 3 only

(d) 1, 2 and 3

Exp: This new bill amends the Maternity Benefit Act of 1961. Under the new provision, women are entitled to visit creches four times a day. Maternity benefit of 26 weeks are provided. Out of 26 weeks, Up to eight weeks can be claimed before delivery. However, one can instead take the entire 26 weeks of leave after the delivery. If the woman has more than two surviving children, the maternity benefit is for 12 weeks only. 

Source: https://www.livemint.com/money/personal-finance/the-benefits-women-are-entitled-to-and-the-rights-they-can-claim-under-maternity-1557655754106.html

Covered in Drishti: https://www.drishtiias.com/daily-updates/daily-news-editorials/maternity-benefits-and-workplace-equality-in-india

77. Which one of the following is not a sub-index of the World Bank’s 'Ease of Doing Business Index'?

(a) Maintenance of law and order

(b) Paying taxes

(c) Registering property

(d) Dealing with construction permits

Exp: Sub Index of Ease of Doing Buisness Index comprises starting a buisness, Getting a location(labour market regulation,constuction permits, getting electrictiy, registering property), Accessing finance (Getting credit, Protecting Minority investor), Dealing with day to day operation (Paying taxes, enforcing contracts).

Source: https://www.worldbank.org/content/dam/doingBusiness/media/Annual-Reports/English/DB2019-report_web-version.pdf

Covered in Drishti: https://www.drishtiias.com/daily-updates/daily-news-analysis/world-banks-ease-of-doing-business-report-2019

78. In India, ‘extend producer responsibility’ was introduced as an important feature in which of the following?

(a) The Bio-medical Waste (Management and Handling) Rules, 1998

(b) The Recycled Plastic (Manufacturing and Usage) Rules, 1999

(c) The e-Waste (Management and Handling) Rules, 2011

(d) The Food Safety and Standard Regulations, 2011

Exp: The E-waste Rules place main responsibility of e-waste management on the producers of the electrical and electronic equipment by introducing the concept of “extended producer responsibility” (EPR). The producers are given a significant responsibility—financial or physical—for the treatment or disposal of post-consumer products.The e-waste (Management & Handling) Rules, 2011 have been notified in May 2011 and are effective from 01-05-2012.

Source: http://cpcb.nic.in/displaypdf.php?id=aHdtZC9JbXBsaW1lbnRhdGlvbkUtV2FzdGUucGRm

Covered in Drishti: https://www.drishtiias.com/daily-updates/daily-news-editorials/the-afterlife-of-e-goods

79. The economic cost of food grains to the Food Corporation of India is Minimum Support Price and bonus (if any) paid to the farmers plus

(a) transportation cost only

(b) interest cost only

(c) procurement incidentals and distribution cost

(d) procurement incidentals and charges for godowns

Exp: The economic cost of foodgrains consists of three components, namely the MSP including central bonus, if applicable, as the price paid to farmers, procurement incidentals, and the cost of distribution.

Source: https://frontline.thehindu.com/static/html/fl1706/17060980.htm

80. In the context of any country, which one of the following would be considered as part of its social capital?

(a) The proportion of literates in the population

(b) The stock of its buildings, other infrastructure and machines

(c) The size of population in the working age group

(d) The level of mutual trust and harmony in the society

Exp: Social capital refers to connections among individuals – social networks and the norms of reciprocity and trustworthiness that arise from them. 

Source: https://darpg.gov.in/sites/default/files/Social_Capital9.pdf

https://economictimes.indiatimes.com/definition/social-capital

81. With reference to the Constitution of India, consider the following statements:

  1. No High Court shall have the jurisdiction to declare any central law to be constitutionally invalid.
  2. An amendment to the Constitution of India cannot be called into question by the Supreme Court of India.

Which of the statements given above is/are correct?

(a) 1 only

(b) 2 only

(c) Both 1 and 2

(d) Neither 1 nor 2

Exp: The 42nd Amendment Act of 1976 curtailed the judicial review power of high court. It debarred the high courts from considering the constitutional validity of any central law. However, the 43rd Amendment Act of 1977 restored the original position.High Court has the power to invalidate any central law which goes against the ethos of the constitution. Similar is the case with Supreme Court. If any constitutional amendment goes against the basic structure of the constitution, Supreme court has the power to call it into question.In 2015, the Supreme Court declared both the 99th Constitutional Amendment, 2014 and the National Judicial Appointments Commission (NJAC) Act, 2014 as unconstitutional and null and void.

82. Consider the following statements:

  1. Purchasing Power Parity (PPP) exchange rates are calculated by comparing the prices of the same basket of goods and services in different countries.
  2. In terms of PPP dollars, India is the sixth largest economy in the world.

Which of the statements given above is/are correct?

(a) 1 only

(b) 2 only

(c) Both 1 and 2

(d) Neither 1 nor 2

Exp: PPPs measure the total amount of goods and services that a single unit of a country’s currency can buy in another country. The PPP between countries A and B measures the amount of country A’s currency required to purchase a basket of goods and services in country A as compared to the amount of country B’s currency to purchase a similar basket of goods and services in country B.

Source: http://agricoop.nic.in/sites/default/files/Cwwg%20data%20as%20on%2012.10.2018.pdf

Covered in Drishti: https://www.drishtiias.com/daily-updates/daily-news-analysis/important-facts-for-prelims-july-12-2018

83. With reference to the cultivation of Khalif crops in India in the last five years, consider the following statements:

  1. Area under rice cultivation is the highest.
  2. Area under the cultivation of jowar is more than that of oilseeds.
  3. Area of cotton cultivation is more than that of sugarcane.
  4. Area under sugarcane cultivation has steadily decreased.

Which of the statements given above are correct?

(a) 1 and 3 only

(b) 2, 3 and 4 only

(c) 2 and 4 only

(d) 1, 2, 3 and 4

Exp: 

84. Among the agricultural commodities imported by India, which one of the following accounts for the highest imports in terms of value in the last five years?

(a) Spices

(b) Fresh fruits

(c) Pulses

(d) Vegetable oils

Exp: The import of vegetable oils in India is highest in last five years due to a cut in duty as mandatory under the Malaysia-India Comprehensive Economic Cooperation Agreement (MICECA). Signed in October 2010 with South East Asian countries,

Source: https://www.business-standard.com/article/markets/edible-oil-imports-jump-11-in-dec-may-rise-further-on-duty-cut-119011500879_1.html

Covered in Drishti: http://agricoop.nic.in/sites/default/files/overviewTrade7082015.pdf

85. In the context of polity, which one of the following would you accept as the most appropriate definition of liberty?

(a) Protection against the tyranny of political rulers

(b) Absence of restraint

(c) Opportunity to do whatever one likes

(d) Opportunity to develop oneself fully

Exp: Option d) seems the most appropriate answer because the Preamble of the Indian Constitution provides for Liberty of thought, expression, belief, faith and worship. This can be interpreted as liberty in the context of polity. 

Source: Preamble

86. Which one of the following is not the most likely measure the Government/RBI takes to stop the slide of Indian rupee?

(a) Curbing imports of non-essential goods and promoting exports

(b) Encouraging Indian borrowers to issue rupee denominated Masala Bonds

(c) Easing conditions relating to external commercial borrowing

(d) Following an expansionary monetary policy

Exp: i. Curbing imports of non essential goods will lessen the demand for dollars and promoting export will help in increasing the flow of dollars into the country - helping control rupee depriciation.

ii. The Masala bond is directly pegged to the Indian Currency. If Indian borrowers issues more rupee denominated Masala bonds, this would increase liquidity in the market or increase in the rupee stock against few currencies in the market and this would help in supporting the rupee.

iii.ECB- it is a type of loan in foreign currencies, made by non-resident lenders. Thus easing conditions of ECB's helps in receiving more loans in foreign currencies that would in increase of forex inflow - leading to rupee appreciation.

iv. Expansionary Monetary Policy- A set of policy measures such as increase in money supply by the RBI to stimulate economy. It cannot influece the variations of rupee value.  

Source: https://www.thehindu.com/business/Economy/curb-on-imports-to-bring-rupee-to-68-70-level-department-of-economic-affairs/article25019942.ece 

The Hindu, Economic Times, Business today Explanation: Self

Covered in Drishti: https://www.drishtiias.com/daily-updates/daily-news-editorials/depreciating-rupee

87. Consider the following statements:

The Reserve Bank of India’s recent directives relating to ‘Storage of Payment System Data, popularly known as data diktat, command the payment system providers that

  1. they shall ensure that entire data relating to payment systems operated by them are stored in a system only in India
  2. they shall ensure that the systems are owned and operated by public sector enterprises
  3. they shall submit the consolidated system audit report to the Comptroller and Auditor General of India by the end of the calendar year

Which of the statements given above is/are correct?

(a) 1 only

(b) 1 and 2 only

(c) 3 only

(d) 1, 2 and 3

Exp: The Storage of Payment System Data directive issued by RBI requires all System Providers to ensure that the entire data relating to payment systems operated by them are stored in a system in India only. Also, System Providers shall provide a System Audit Report (SAR) and the audit should be conducted by CERT-IN empaneled auditors.

Source: RBI Website: https://www.rbi.org.in/scripts/NotificationUser.aspx?Id=11244

Covered in Drishti: https://www.drishtiias.com/daily-updates/daily-news-editorials/all-things-internet

88. Which of the following adopted a law on data protection and privacy for its citizens known as General Data Protection Regulation’ in April 2016 and started implementation of it from 25th May, 2018?

(a) Australia

(b) Canada

(c) The European Union

(d) The United States of America

Exp: The General Data Protection Regulation (GDPR) standardizes data protection law across all 28 EU countries and imposes strict new rules on controlling and processing personally identifiable information (PII). It also extends the protection of personal data and data protection rights by giving control back to EU residents. GDPR replaces the 1995 EU Data Protection Directive, and went into force on May 25, 2018.

Source: EUGDPR https://eugdpr.org/

https://www.forbes.com/sites/quora/2018/02/14/what-is-general-data-protection-regulation/#2403931b62dd

Covered in Drishti: https://www.drishtiias.com/daily-updates/daily-news-editorials/all-things-internet

89. Recently, India signed a deal known as ‘Action Plan for Prioritization and Implementation of Cooperation Areas in the Nuclear Field’ with which of the following countries?

(a) Japan

(b) Russia

(c) The United Kingdom

(d) The United States of America

Exp: During the visit of H. E. Vladimir Putin, President of the Russian Federation to India, Action Plan for Prioritization and Implementation of Co-operation Areas in the Nuclear Field Identified Jointly by India and Russia was signed on 5th October, 2018 in New Delhi.

Source: https://dae.nic.in/?q=node/1136

90. The money multiplier in an economy increases with which one of the following?

(a) Increase in the cash reserve ratio

(b) Increase in the banking habit of the population

(c) Increase in the statutory liquidity ratio

(d) Increase in the population of the country

Exp: The money-multiplier is the maximum amount of broad money (M3 Money) that could be created by the commercial banks for a given fixed amount of base money or reserve ratios. Or, simply it can be stated that the maximum amount of money that the banking system generates with each rupees of reserves.

So, more the people will be linked with the banking system or have the habit of banking more will be the money in use in the form of loans, credits etc. that results an increase in economic activity. This will dirctly help in increase money multiplier in an economy. The money multiplier goes up because of the reduction in the cash reserve ratio (CRR). 

Source: https://www.imf.org/external/region/tlm/rr/pdf/laomonetarystatistics.pdf

https://shodhganga.inflibnet.ac.in/bitstream/10603/115659/8/08_chapter%203.pdf

Reference: RBI Website Explanation: Self

91. In the context of digital technologies for entertainment, consider the following statements:

  1. In Augmented Reality (AR), a simulated environment is created and the physical world is completely shut out.
  2. In Virtual Reality (VR), images generated from a computer are projected onto real-life objects or surroundings.
  3. AR allows individuals to be present in the world and improves the experience using the camera of smart-phone or PC.
  4. VR closes the world, and transposes an individual, providing complete immersion experience.

Which of the statements given above is/are correct?

(a) 1 and 2 only

(b) 3 and 4

(c) 1, 2 and 3

(d) 4 only

Exp: Virtual reality (VR) immerses users in a fully artificial digital environment.

Augmented reality (AR) overlays virtual objects on the real-world environment.

Mixed reality (MR) not just overlays but anchors virtual objects to the real world.

Source: https://www.forbes.com/sites/quora/2018/02/02/the-difference-between-virtual-reality-augmented-reality-and-mixed-reality/#5527da952d07

https://www.fi.edu/difference-between-ar-vr-and-mr

Covered in Drishti: https://www.drishtiias.com/daily-updates/daily-news-analysis/virtual-reality-centre-of-indian-navy

92. The word ‘Denisovan’ is sometimes mentioned in media in reference to

(a) fossils of a kind of dinosaurs

(b) an early human species

(c) a cave system found in North-East India

(d) a geological period in the history of Indian subcontinent

Exp: For the first time, scientists have found fossils from an extinct ancient human lineage known as the Denisovans outside of Siberia.

Denisovans were an extinct group of hominins that were close relatives of Neanderthals. They are known primarily from a handful of fossil fragments found at Denisova Cave in Siberia, and from genetic clues that linger in the DNA of people across Asia.

Source: Nature. com https://www.nature.com/articles/d41586-019-00672-2

Covered in Drishti: https://www.drishtiias.com/daily-updates/daily-news-analysis/important-facts-for-prelims-3rd-may-2019

93. With reference to the recent developments in science, which one of the following statements is not correct?

(a) Functional chromosomes can be created by joining segments of DNA taken from cells of different species.

(b) Pieces of artificial functional DNA can be created in laboratories.

(c) A piece of DNA taken out from an animal cell can be made to replicate outside a living cell in a laboratory.

(d) Cells taken out from plasma and animals can be made to undergo cell division in laboratory petri dishes.

Exp: In a major step toward creating artificial life, US researchers have developed a living organism that incorporates both natural and artificial DNA and is capable of creating entirely new, synthetic proteins. Hence, option (b) is correct. A wide variety of double-stranded DNA templates are replicated extensively in an in vitro DNA replication system containing the purified proteins specified by seven T4 bacteriophage DNA replication genes (32, 41, 43, 44, 62, 45, and 61). Hence, option (c ) is correct. Through micropropagation plant can be developed in laboratory and chlamydomnas cells can be replicated in culture through light variations. Hence, option (d) is correct.

Source: http://ncert.nic.in/ncerts/l/lebo109.pdf

Covered in Drishti: https://www.independent.co.uk/news/science/artificial-life-synthetic-dna-scientists-living-organisms-create-scripps-research-institute-floyd-a8083966.html

94. Consider the following statements:

A digital signature is

  1. an electronic record that identifies the certifying authority issuing it
  2. used to serve as a proof of identity of an individual to access information or server on Internet
  3. an electronic method of signing an electronic document and ensuring that the original content is unchanged

Which of the statements given above is/are correct?

(a) 1 only

(b) 2 and 3 only

(c) 3 only

(d) 1, 2 and 3

Exp: If we consider Digital Signature same as Digital Signature Certificate then option should be d). The certificate contains information about the user’s identity (name, pin code, country, email address, the date the certificate was issued and the name of the certifying authority). However, if they are considered as different, then the answer should be c).

Source: https://www.independent.co.uk/news/science/artificial-life-synthetic-dna-scientists-living-organisms-create-scripps-research-institute-floyd-a8083966.html

http://www.jbc.org/content/255/9/4290.full.pdf

95. In the context of wearable technology, which of the following tasks is/are accomplished by wearable devices?

  1. Location identification of a person
  2. Sleep monitoring of a person
  3. Assisting the hearing impaired person

Select the correct answer using the code given below:

(a) 1 only

(b) 2 and 3 only

(c) 3 only

(d) 1, 2 and 3

Exp: In recent times, there has been a new surge in wearable technology worn on the ear, referred to as hearables. Wearable technology is a category of electronic devices that can be worn as accessories, embedded in clothing, implanted in the user's body, or even tattooed on the skin.

Source: https://www.investopedia.com/terms/w/wearable-technology.asp

96. 'RNA interference (RNAi)' technology has gained popularity in the last few years. Why?

  1. It is used in developing gene silencing therapies.
  2. It can be used in developing therapies for the treatment of cancer.
  3. It can be used to develop hormone replacement therapies.
  4. It can be used to produce crop plants that are resistant to viral pathogens.

Select the correct answer using the code given below.

(a) 1, 2 and 4

(b) 2 and 3

(c) 1 and 3

(d) 1 and 4 only

Exp: RNA interference (RNAi), regulatory system occurring within eukaryotic cells (cells with a clearly defined nucleus) that controls the activity of genes. RNAi functions specifically to silence, or deactivate, genes. RNAi is being explored as a form of treatment for a variety of diseases, including macular degeneration, hepatitis, AIDS, Huntington disease, and cancer. RNAi is used as a natural defense mechanism against molecular parasites such as jumping genes and viral genetic elements.

Source: https://www.britannica.com/science/RNA-interference

97. Recently, Scientists observed the merger of giant blackholes’ billions of light-years away from the Earth. What is the significance of this observation?

(a) 'Higgs boson particles' were detected.

(b) 'Gravitational waves' were detected.

(c) Possibility of inter-galactic space travel through 'wormhole' was confirmed.

(d) It enabled the scientists to understand 'singularity'.

Exp: When the two supermassive black holes in each of these systems finally come together in millions of years, their encounters will produce strong gravitational waves. Gravitational waves produced by the collision of two stellar-mass black holes have already been detected by the Laser Interferometer Gravitational-Wave Observatory (LIGO). Observatories such as the planned NASA/ESA space-based Laser Interferometer Space Antenna (LISA) will be able to detect the lower-frequency gravitational waves from supermassive black-hole mergers, which are a million times more massive than those detected by LIGO

Covered in Drishti: https://www.drishtiias.com/infographics/info-paper3/gravitational-waves

98. Which of the following are the reasons for the occurrence of multi-drug resistance in microbial pathogens in India?

  1. Genetic predisposition of some people
  2. Taking incorrect doses of antibiotics to cure diseases
  3. Using antibiotics in livestock farming
  4. Multiple chronic diseases in some people

Select the correct answer using the code given below.

(a) 1 and 2

(b) 2 and 3 only

(c) 1, 3 and 4

(d) 2, 3 and 4

Exp: Antimicrobial resistance is not through genetic predisposition rather it occurs naturally over time, usually through genetic changes. However, the misuse and overuse of antimicrobials is accelerating this process. In many places, antibiotics are overused and misused in people and animals, and often given without professional oversight. Examples of misuse include when they are taken by people with viral infections like colds and flu, and when they are given as growth promoters in animals or used to prevent diseases in healthy animals.

Source: WHO, Niaid.nih.gov

Covered in Drishti: https://www.drishtiias.com/to-the-points/paper3/to-the-point-paper-3-antimicrobial-resistance-amr

99. What is Cas9 protein that is often mentioned in news?

(a) A molecular scissors used in targeted gene editing

(b) A biosensor used in the accurate detection of pathogens in patients

(c) A gene that makes plants pest-resistant

(d) A herbicidal substance synthesized in genetically modified crops

Exp: CRISPR Cas9 is an acronym for “Clustered Regularly Interspaced Short Palindromic Repeats.” CRISPR genome engineering technology enables scientists to easily and precisely edit the DNA of any genome.

Source: https://www.synthego.com/learn/crispr

Covered in Drishti: Tarkash Vol II Pg 146

https://www.drishtiias.com/daily-updates/daily-news-analysis/worlds-first-gene-edited-babies

100. Which one of the following statements is not correct?

(a) Hepatitis B virus is transmitted much like HIV.

(b) Hepatitis B. unlike Hepatitis C, does not have a vaccine.

(c) Globally, the number of people infected with Hepatitis B and C viruses arc several times more than those infected with HIV.

(d) Some of those infected with Hepatitis B and C viruses do not show the symptoms for many years.

Exp: The hepatitis B vaccine is a safe and effective vaccine that is recommended for all infants at birth and for children up to 18 years. The hepatitis B vaccine is also known as the first “anti-cancer” vaccine because it prevents hepatitis B, the leading cause of liver cancer worldwide.

Source: https://www.hepb.org/prevention-and-diagnosis/vaccination/